Sie sind auf Seite 1von 54

July 2010

MEE Questions
and Analyses

Copyright © 2010 by the National Conference of Bar Examiners.


All rights reserved.

July 2010 MEE Questions and Analyses

Contents

Preface............................................................................................................................................ iii

Description of the MEE ................................................................................................................. iii

Instructions..................................................................................................................................... iv

JULY 2010 QUESTIONS

Agency and Partnership Question....................................................................................................3

Contracts and Negotiable Instruments Question..............................................................................4

Decedents’ Estates Question............................................................................................................5

Constitutional Law Question ...........................................................................................................6

Real Property Question ....................................................................................................................7

Family Law Question.......................................................................................................................8

Federal Civil Procedure Question....................................................................................................9

Criminal Law Question..................................................................................................................11

Corporations Question ...................................................................................................................12

JULY 2010 ANALYSES

Agency and Partnership Analysis ..................................................................................................15

Contracts and Negotiable Instruments Analysis ............................................................................17

Decedents’ Estates Analysis ..........................................................................................................22

Constitutional Law Analysis..........................................................................................................26

Real Property Analysis ..................................................................................................................29

i
Contents

Family Law Analysis .....................................................................................................................33

Federal Civil Procedure Analysis ..................................................................................................37

Criminal Law Analysis ..................................................................................................................41

Corporations Analysis....................................................................................................................44

ii
Preface

The Multistate Essay Examination (MEE) is developed by the National Conference of Bar
Examiners (NCBE). This publication includes the questions and analyses from the July 2010
MEE. Each test includes nine questions; most jurisdictions that use the MEE select six questions
for their applicants to answer. In the actual test, the questions are simply numbered rather than
being identified by area of law. The instructions for the test appear on page iv. For more
information, see the MEE Information Booklet, available on the NCBE website at www.ncbex.org.

The model analyses for the MEE are illustrative of the discussions that might appear in excellent
answers to the questions. They are provided to the user jurisdictions for the sole purpose of
assisting graders in grading the examination. These model analyses are not an official grading
guide. Some states grade the MEE on the basis of state law, and jurisdictions are free to modify
the analyses, including the suggested weights given to particular points, as they wish. Grading of
the MEE is the exclusive responsibility of the jurisdiction using the MEE as part of its
admissions process.

The topics covered by each question are listed on the first page of its accompanying analysis,
followed by roman numerals which refer to the MEE subject matter outline for that topic. For
example, the Federal Civil Procedure question on the July 2010 MEE tested the following areas
from the Federal Civil Procedure outline: I.A.1., jurisdiction and venue—subject matter
jurisdiction—federal courts, and I.B., jurisdiction and venue—jurisdiction over parties. Subject
matter outlines are included in the MEE Information Booklet.

Description of the MEE

The MEE is a series of essay questions, any of which a jurisdiction may select to include as a part
of its bar examination. Applicants are expected to spend approximately 30 minutes answering
each MEE question administered. The areas of law that may be covered by the questions on any
MEE are Business Associations (Agency and Partnership; Corporations and Limited Liability
Companies), Conflict of Laws, Constitutional Law, Contracts, Criminal Law and Procedure,
Evidence, Family Law, Federal Civil Procedure, Real Property, Torts, Trusts and Estates
(Decedents’ Estates; Trusts and Future Interests), and Uniform Commercial Code (Negotiable
Instruments [Commercial Paper]; Secured Transactions). Some questions may include issues in
more than one area of law.

The purpose of the MEE is to test the applicant’s ability to (1) identify legal issues raised by a
hypothetical factual situation; (2) separate material which is relevant from that which is not; (3)
present a reasoned analysis of the relevant issues in a clear, concise, and well-organized
composition; and (4) demonstrate an understanding of the fundamental legal principles relevant
to the probable solution of the issues raised by the factual situation. The primary distinction
between the MEE and the Multistate Bar Examination (MBE) is that the MEE requires the
applicant to demonstrate an ability to communicate effectively in writing.

iii
Instructions
The back cover of each test form contains the following instructions:

Do not break the seal on this booklet until you are told to begin.

Each question is designed to be answered in 30 minutes. There will be no break once the
formal testing session begins. You may answer the questions in any order you wish. Do
not answer more than one question in each answer booklet. If you make a mistake or
wish to revise your answer, simply draw a line through the material you wish to delete.

If you are using a laptop computer to answer the questions, your jurisdiction will provide
you with specific instructions to follow.

Read each fact situation very carefully and do not assume facts that are not given in the
question. Do not assume that each question covers only a single area of the law; some of
the questions may cover more than one of the areas you are responsible for knowing.

Demonstrate your ability to reason and analyze. Each of your answers should show an
understanding of the facts, a recognition of the issues included, a knowledge of the
applicable principles of law, and the reasoning by which you arrive at your conclusion.
The value of your answer depends not as much upon your conclusions as upon the
presence and quality of the elements mentioned above.

Clarity and conciseness are important, but make your answer complete. Do not volunteer
irrelevant or immaterial information.

Your jurisdiction may instruct you to answer MEE questions according to the law of the
jurisdiction. Absent such an instruction, you should answer the questions by applying
fundamental legal principles rather than local case or local statutory law.

iv
July 2010
Questions
Agency and Partnership Question

On January 2, Fran opened Petals, a floral shop, and operated it as a sole proprietorship. Petals
soon ran into financial difficulties, and Fran could not pay its bills.

On March 1, Fran asked her friends Gina and Hank for financial help.

On April 1, in response to Fran’s request, Gina delivered a check payable to “Petals” to Fran. In
exchange for this contribution, Fran agreed to pay Gina 25% of the monthly net profits of Petals
for as long as Petals remained in business. Gina also agreed that, if Petals suffered losses, she
would share those losses with Fran.

Gina began working at the shop with Fran and helped Fran with business planning for Petals.

On April 2, also in response to Fran’s request, Hank delivered a check payable to “Petals” to
Fran and noted on the memo line of the check “loan to Petals.” Hank agreed to accept 25% of the
monthly net profits of Petals until his loan (plus interest) was repaid in full.

Fran used the proceeds of the checks from Gina and Hank to purchase equipment, supplies, and a
delivery truck in the name of Petals.

Beginning in April, on the last day of each month, Fran distributed to both Gina and Hank 25%
of Petals’ monthly net profits.

On September 1, Gina wrote a letter to her son Ivan stating that she was assigning to Ivan, as a
gift, all of her interest in Petals effective immediately. Gina gave a copy of that letter to Fran.
Fran told Gina, “I don’t want anything to do with Ivan.” Gina continued to be active in the
business operations of Petals.

On September 30, Fran distributed the monthly net profits of Petals to Gina and Hank, but
distributed nothing to Ivan.

On October 10, Ivan demanded that Fran distribute Gina’s share of Petals’ net profits to him and
that she also allow him to inspect the books and records of Petals.

On October 15, Gina learned that Fran was using Petals’ delivery truck on Sundays to transport
her children to their soccer games. Gina demanded that Fran stop doing so, but Fran refused,
noting that the truck was not being used for Petals’ business on Sundays.

1. What is the legal relationship among Fran, Gina, and Hank? Explain.

2. Is Ivan entitled to Gina’s share of the monthly net profits of Petals? Explain.

3. Is Ivan entitled to inspect the books and records of Petals? Explain.

4. Is Fran entitled to use the delivery truck on Sundays to take her children to their soccer
games? Explain.

3
Contracts and Negotiable Instruments Question

Buyer, who was in the market for a car, heard that Seller wanted to sell his car for $5,000. On
June 1, Buyer visited Seller and saw the car. Buyer asked Seller about the car’s condition. In
response, Seller said, “The car is in tip-top shape—the brakes and clutch were replaced in the last
six months. It’s in beautiful shape for a vehicle of this age. Good for another 100,000 miles
easy.”

Seller agreed to sell the car to Buyer for $5,000. They both signed the following document:
“Seller agrees to sell, and Buyer agrees to buy, Seller’s car for the price of $5,000. Buyer will
pick up the car at Seller’s home on June 2 and pay Seller $4,000 in cash and give Seller a check
for $1,000 at that time.”

On June 2, Buyer came to Seller’s home. Before handing the check to Seller, Buyer said, “I’d
like my mechanic to look at the car to make sure that it is as you represented it.” Seller
responded, “Don’t waste money on a mechanic. The car is exactly as I described it.” Even
though Buyer, while at Seller’s home, had no way to tell if the brakes and clutch were as
represented, Buyer thought that it would be a waste of time and money to visit a mechanic and
thus decided to proceed with the transaction. Accordingly, after briefly inspecting the car, Buyer
gave Seller $4,000 in cash and a $1,000 check drawn on First Bank. Seller handed Buyer the
keys to the car, and Buyer left with the car.

On June 3, Seller went to Checkco, where he indorsed the back of Buyer’s check by signing his
name with no other words and handed the check to the clerk in exchange for $950 in cash.
Checkco was unaware of any facts about the transaction that gave rise to the check.

On June 10, the car broke down and Buyer had it towed to a mechanic’s shop. After looking at
the car, the mechanic accurately told Buyer that the clutch had failed because it was old and
needed to be replaced. The mechanic also warned Buyer that the brakes were unsafe and that the
engine needed a complete overhaul or it wouldn’t last another 10,000 miles. The mechanic told
Buyer that if the car had been as represented by Seller, it would have had a market value of
$5,000, but in its current condition the car was worth only about $500—its value as salvage for
parts.

On June 11, Buyer hand-delivered a letter to Seller. The letter informed Seller that Buyer was
revoking his acceptance of the car and that Seller could recover his car at the mechanic’s shop.
Buyer also visited First Bank and instructed it to refuse to pay the check that Buyer had given
Seller.

On June 12, an agent of Checkco went to First Bank and demanded payment of Buyer’s check.
First Bank refused to pay the check because of Buyer’s instruction and gave the check back to
the agent. Checkco then promptly gave notice of First Bank’s refusal to pay to both Seller and
Buyer.

1. What rights, if any, does Buyer have against Seller? Explain.

2. What rights, if any, does Checkco have against Buyer and against Seller? Explain.

4
Decedents’ Estates Question

Three years ago, Testator told his attorney to draft a will leaving $20,000 to his Sister and the
balance of his estate to his children. Testator told his attorney that he was divorced and that he
had two children, Abby and Bruce, both biological children born during Testator’s first marriage.
Testator did not tell the attorney that he had adopted his stepchild, Carl, when Carl was two
years old.

The attorney prepared a typed will based on Testator’s instructions. When Testator came to the
attorney’s office to execute the will, the attorney placed the three unstapled pages of the will on
her desk and said to Testator, “Please sign your will.”

Page 1 of the will included introductory clauses and two bequests reading: “I give $20,000 to
Sister. I give the balance of my estate to my children, in equal shares.” Pages 2 and 3 of the will
contained clauses relating to the responsibilities of the executor. At the bottom of page 3, there
were lines for Testator’s and his witnesses’ signatures and an attestation clause.

Testator, in the presence of two witnesses, read the three pages, declared the document on the
attorney’s desk to be his will, and signed it on the line provided on page 3. Both witnesses, who
were able to see all three pages, signed their names underneath Testator’s signature and again
under the attestation clause. The attorney thereafter folded the three pages together and gave
them to Testator.

Two years ago, Testator decided that he wanted to leave more money to Sister. To accomplish
this, Testator crossed out the bequest reading “I give $20,000 to Sister” on page 1 of the will and
wrote above the crossed-out phrase, “I give $40,000 to Sister.”

Last month, Testator died. Testator’s will, all three pages folded together, was found in a night
table in Testator’s bedroom. Abby, Bruce, Carl, and Sister survived Testator. In addition, Don
claims to be Testator’s nonmarital child.

The only relevant state statutes provide:

I. “No will or codicil thereto is valid unless signed by the testator and two attesting
witnesses.”

II. “A will may be revoked, in whole or in part, by destruction or cancellation.”

III. “If a decedent dies intestate survived by children and no spouse, the decedent’s
entire estate passes to his children, in equal shares.”

1. Is Testator’s will valid? Explain.

2. To whom should Testator’s estate be distributed? Explain.

5
Constitutional Law Question

The Church of Peace (the Church) is a religious organization that advocates “peace to everyone.”
Recently, a Church chapter (Chapter) was organized in the town of Homestead. Chapter
members decided to spread the Church’s message to the people of Homestead by handing out
leaflets that proclaimed in bold letters, “PEACE TO ALL!” Chapter members who participated
in passing out the leaflets stood on a public sidewalk and distributed the leaflets to pedestrians.
The Chapter members did not block traffic or take any actions except passing out leaflets and
remarking, “Peace to all!”

Many people who took the leaflets threw them onto the sidewalk, and Homestead employees
spent several hours cleaning up these discarded leaflets. Chapter was fined $3,000 under a
municipal anti-leafleting ordinance that prohibits any distribution of leaflets “in or on any public
space, including roads, streets, and sidewalks.” No Chapter member threw leaflets or other litter
onto the ground.

Chapter members who attend High School, a public school in Homestead, recently formed the
“Church of Peace Club” (Church Club) to pray together and to do good works. High School has
a policy that permits student groups to meet in High School classrooms after scheduled classes.
Under this policy, student groups must first obtain permission from Principal before using a
classroom for a meeting. Pursuant to this policy, the Chess Club, the Drama Club, and the Future
Lawyers Club all use classrooms for after-school meetings. Church Club officers asked Principal
if they could meet in a classroom after school. Principal denied this request and stated that after-
school use of a classroom by Church Club would be “a violation of the separation of church and
state.”

Father, a Chapter member and the parent of a Church Club officer, learned about Principal’s
decision and went to High School to see Principal. Outside Principal’s office was a sign reading
“No admittance without an appointment.” Father, who had no appointment, threw open the
closed door and marched into Principal’s office, interrupting a meeting between Principal and
another parent, and told Principal, “Your policy is unwise and unconstitutional. I believe that you
are discriminating against members of my faith.” Principal asked Father to leave the office until
the meeting with the other parent was concluded, but Father refused. Principal called the police,
who forcibly removed Father from Principal’s office.

Father was convicted of trespassing on government property.

Does the First Amendment, as applied to state and local governments through the Fourteenth
Amendment,

1. Preclude Homestead’s enforcement of its anti-leafleting ordinance against Chapter?


Explain.

2. Preclude Principal’s denial of Church Club’s request to use classroom space for its
meetings? Explain.

3. Provide grounds to vacate Father’s trespass conviction? Explain.

6
Real Property Question

Eighty years ago, Owner, the owner of vacant land known as Blackacre, conveyed Blackacre to a
local school district (School) “if School uses Blackacre only to teach children aged 5 to 13.”
Shortly after acquiring title to Blackacre, School erected a classroom building on Blackacre and
began teaching children aged 5 to 13 in that building.

Seventy years ago, Owner died and left his entire estate to Daughter.

School used the classroom building to teach its students aged 5 to 13 until three years ago when,
due to increasing enrollments, School built a new classroom building three miles from Blackacre
and converted the classroom building on Blackacre into administrative offices.

The building on Blackacre is now exclusively occupied by administrative offices, and all School
students aged 5 to 13 are taught in the new classroom building.

Two years ago, Daughter died. Daughter did not object to School’s altered use of Blackacre
before her death. She devised her entire estate to her Husband for life, with the remainder to “my
surviving children.” Daughter was survived by Husband and two children, Ann and Bill.

One year ago, Bill died. Bill’s entire estate passed to his wife, Mary.

One month ago, Husband, the life tenant under Daughter’s will, died. Husband was survived by
Ann and by Bill’s widow, Mary.

State law provides:

I. “Actions to recover the possession of real property shall be brought within 10


years after the cause of action accrues.”

II. “All future interests are alienable, devisable, and descendible to the extent they do
not expire as a result of the holder’s death.”

III. “Conditions and limitations in a deed shall not be construed as covenants.”

There are no other relevant statutes.

What interests, if any, do School, Ann, and Mary have in Blackacre? Explain.

7
Family Law Question

Fifteen years ago, Husband and Wife married in State A. At the time of the marriage, Husband
and Wife were both 35 years old, and each had a high school education. Wife worked as an
administrative assistant for a manufacturing company, where she had been employed for six
years. Husband was unemployed.

Husband was a spendthrift with a history of selling his possessions to fund unsuccessful business
ventures. Aware of his personal failings, Husband asked Wife to enter into a premarital
agreement under which (a) Wife would become the sole owner of all assets owned by Husband
before his marriage to Wife; (b) Wife would pay all of Husband’s premarital debts; (c) if
Husband and Wife divorced, Wife would have exclusive rights to all assets acquired by either
Husband or Wife during their marriage; and (d) if Husband and Wife divorced, both would
waive all claims to alimony. Wife agreed to Husband’s proposal. Husband and Wife both
disclosed their assets to each other.

Attorney thereafter prepared a premarital agreement based on Husband’s and Wife’s


understanding. Husband and Wife signed the agreement one week after a meeting at which
Attorney explained the consequences of signing the agreement. The agreement contained a
provision stating that both Husband and Wife had chosen to forgo individual representation by a
lawyer. After signing the agreement, Husband transferred title to his assets to Wife.

Throughout the marriage, Wife has performed virtually all household chores. Wife has also
worked full time at the manufacturing company; at times, she has also held a part-time job to pay
for household expenses. Wife paid Husband’s premarital debts with her earnings and with some
of the assets Husband had transferred to her. She sold the balance of the assets Husband had
transferred to her to pay for flying lessons for Husband. Husband got a commercial pilot’s
license after taking these lessons but never tried to find work as a pilot. Instead, Husband worked
at part-time odd jobs until five years ago, when he was injured in a car accident. Thereafter,
Husband claimed he was disabled and ceased working altogether. Wife currently earns $35,000
per year.

Wife has sued Husband for divorce in State A, which terms spousal support “alimony.” Husband
claims that he is entitled to alimony and a share of the couple’s assets. Wife claims that she is
entitled to all assets titled in her name and in Husband’s name. These assets are (a) the marital
home, purchased by Wife during the marriage and titled in Wife’s name; (b) Wife’s employment
pension; and (c) real estate inherited by Husband during the marriage and titled in Husband’s
name.

Wife has asked your law firm these questions:

1. Is the premarital agreement enforceable? Explain.

2. If the premarital agreement is unenforceable, what assets are divisible at divorce?


Explain.

3. If the premarital agreement is unenforceable, is Husband entitled to alimony? Explain.

8
Federal Civil Procedure Question

Until recently, Paul had always lived in State A. Last year, he decided he would move to State B
for at least one year and, after a year, decide whether to remain in State B or return to State A.
Six months ago, Paul moved to State B, rented an apartment, and took a job as a temporary
employee. Paul has enjoyed living in State B so much that he recently left his temporary job and
accepted a position as a permanent employee at a law firm in State B.

Shortly after he moved to State B, Paul bought a vacation home in State A, which he visits about
once a month for two or three days. To pay for the vacation home, Paul obtained a loan from
Credit Union in State A. Credit Union is incorporated in and chartered by State A. Its only office,
located in State A, is both its corporate headquarters and the place where it transacts business
with its customers. Ninety-five percent of Credit Union’s customers are State A residents who do
business with Credit Union in person at its State A office.

Paul’s loan agreement with Credit Union provides that he will repay the loan in monthly
installments over a 30-year period. Credit Union has a mortgage on Paul’s vacation home to
secure the debt. The loan paperwork lists Paul’s State B address as his mailing and home
address. The loan agreement also contains a privacy provision whereby Credit Union agrees not
to disclose Paul’s personal information to any third party without Paul’s written permission.
Credit Union sends a loan statement and payment coupon to Paul’s State B address each month,
and Paul returns the payment coupon with a check for the payment amount.

After the loan closed, a Credit Union employee mailed copies of all the loan paperwork to Paul.
Unfortunately, the employee misread Paul’s address in State B and sent the paperwork to an
incorrect address. Several months later, Paul discovered that someone had gotten his loan
paperwork and had used the information (including Paul’s Social Security number and credit
card numbers) to steal his identity. The identity thief had quickly accumulated $150,000 in
unpaid bills in Paul’s name. Paul’s credit rating was ruined, and no one would extend him new
credit.

Paul has sued Credit Union in the United States District Court for the District of State B for
breach of the privacy provisions of the loan contract. The parties have stipulated that Paul’s
actual loss was $80,000. Paul’s suit seeks $240,000 in damages, plus attorney’s fees, pursuant to
a State A statute that entitles victims of identity theft to recover treble damages and attorney’s
fees from anyone who wrongfully discloses their personal information. Paul’s complaint also
asserts that a federal statute restricting damages in state-law identity-theft cases to actual
damages is unconstitutional and therefore does not preempt the treble damages provisions of the
State A statute. The complaint asserts that the State B federal court has both diversity and
federal-question jurisdiction over the case.

The long-arm statute of State B extends personal jurisdiction as far as the Constitution allows.

1. May the United States District Court for the District of State B exercise personal
jurisdiction over Credit Union? Explain.

9
Federal Civil Procedure Question

2. Does the United States District Court for the District of State B have diversity jurisdiction
over the case? Explain.

3. Does the United States District Court for the District of State B have federal-question
jurisdiction over the case? Explain.

10
Criminal Law Question

Customer went to Star Computers (Star) to buy a refurbished computer. Upon arrival, Customer
was approached by Owner, who identified himself as the owner of Star. Owner directed
Customer to a refurbished desktop computer and told Customer, “We have the best refurbished
computers in town. We send used computers to a computer technician who always installs new
hard drives and replaces any defective parts.” Owner made these claims because Owner believed
that they would be effective in persuading Customer to buy a refurbished computer. In fact,
Customer was persuaded by Owner’s claims and purchased a computer for $250 cash.

At the time of this transaction, Owner did not believe that Star had the best refurbished
computers in town. Owner was aware of at least two other computer stores in town and believed
that the refurbished computers sold by these other stores were better than those sold by Star.
Owner also thought it was very likely that the computer technician used by Star did not actually
install new hard drives in the refurbished computers. Owner had never raised the issue with the
technician because the technician offered much faster service and lower rates than those of any
other technician in the area.

After Customer’s purchase, a local news station conducted an investigation into the computer
technician used by Star and reported that the technician did not install new hard drives in any of
the computers she refurbished. After the report aired, the computer technician acknowledged that
no new hard drives had been installed in the computers she had refurbished for Star.

Owner has been charged with larceny by false pretenses in connection with the computer sale to
Customer.

Is Owner guilty of larceny by false pretenses? Explain.

11
Corporations Question

On December 30, X Corporation’s legal record date, X Corporation had 100 shares of issued and
outstanding common stock. Fifty shares were owned by Amy, 25 shares were owned by Brian,
and 25 shares were owned by Carter. X Corporation also had 50 shares of stock that it previously
had issued to, but later repurchased from, Amy.

On January 30, X Corporation’s annual shareholders’ meeting was validly held. Before the
meeting, X Corporation’s staff prepared a list of shareholders entitled to vote at the meeting and
mailed proper notice to them. That notice stated that a proposal requiring shareholder approval
would be voted on at the annual shareholders’ meeting.

Before the annual shareholders’ meeting and in a timely manner, Amy mailed in her duly
executed proxy, directing the secretary of X Corporation to vote her 50 shares in favor of the
proposal. However, before the annual shareholders’ meeting date, Zach called the secretary of
X Corporation and truthfully told the secretary that Amy’s shares belonged to Zach because he
had bought the shares from Amy on December 31. Zach then mailed the secretary a duly
executed proxy directing the secretary of X Corporation to vote his 50 shares against the
proposal.

Prior to the annual shareholders’ meeting, Brian duly executed a proxy in favor of Dell. The
proxy stated in its entirety, “I, Brian, hereby grant Dell full authority to vote my 25 shares of
X Corporation at the January 30th annual shareholders’ meeting.” Dell timely mailed a duly
executed proxy directing the secretary of X Corporation to vote Brian’s 25 shares against the
proposal. Dell also sent the secretary a copy of the proxy given to Dell by Brian. Brian, however,
attended the annual meeting and voted his 25 shares in favor of the proposal.

Carter personally appeared at the annual shareholders’ meeting and voted his 25 shares against
the proposal.

X Corporation’s president attended the annual meeting and, on behalf of X Corporation, voted
the 50 shares that X Corporation had repurchased from Amy against the proposal.

X Corporation’s Articles of Incorporation require an affirmative vote by the holders of two-


thirds of the shares entitled to be voted to approve any proposal at a shareholders’ meeting. The
bylaws, on the other hand, require a unanimous vote of such shares to approve any proposal.

Your law firm represents X Corporation. You have been asked to advise the firm’s senior partner
on whether the proposal received sufficient votes to be approved. Explain your conclusion.

12
July 2010
Analyses
Agency and Partnership Analysis

(Agency and Partnership V.A.; VI.; VII.A. & B.)

Legal Problems: (1) What is the legal relationship among Fran, Gina, and Hank?

(2) Is Ivan entitled to Gina’s share of the monthly net profits of Petals?

(3) Is Ivan entitled to inspect the books and records of Petals?

(4) Is Fran entitled to use the delivery truck on Sundays to take her
children to their soccer games?

DISCUSSION

Summary

Fran and Gina are general partners in the Petals partnership, but Hank is only a creditor
of Petals. As a transferee of Gina’s right to receive net profits in Petals, Ivan is legally entitled to
Gina’s share of those profits. However, that assignment does not make Ivan a partner. Therefore,
Ivan does not have a right to inspect the books and records of Petals. Fran is not entitled to use
the delivery truck on Sundays to take her children to their soccer games.

Point One (35%)


Fran and Gina are partners in Petals. Hank is a creditor of Petals.

A partnership is defined as “the association of two or more persons to carry on as co-


owners a business for profit . . . whether or not the persons intend to form a partnership.”
UNIFORM PARTNERSHIP ACT (UPA) (1997) § 202(a); see also UPA (1914) § 6(1). Here, a
partnership was formed when Gina joined Fran in running Petals. Because the partnership was
formed without the formalities required of other types of partnerships, it is a general partnership.
Fran and Gina each receive a share of the net profits of Petals, and each is therefore
presumed to be a partner in the business. UPA (1997) § 202(c)(3); UPA (1914) § 7(4). This
presumption is supported by the fact that Gina is to receive her share of the net profits “for as
long as Petals remains in business,” and that Fran and Gina each work in the business and
participate in the business planning for Petals. Further, Gina agreed to share in any losses that
Petals might suffer. Therefore, Fran and Gina are partners in the Petals partnership.
Hank also receives a share of the net profits of Petals and, therefore, also might be
presumed to be a partner in the partnership. However, this presumption does not apply where
those profits are “received in payment of a debt by installment or otherwise.” UPA (1997)
§ 202(c)(3)(i); UPA (1914) § 7(4)(a). The statement on the memo line of Hank’s check is
evidence that Hank loaned money to the partnership. Hank is entitled to receive a share of the net
profits of Petals only until his loan (plus interest) to Petals is paid in full, and therefore he is not
presumed to be a partner in the business. In addition, Hank did not agree to share in any losses

15
Agency and Partnership Analysis

that Petals might suffer. Further, Hank does not play any role in the day-to-day operations of
Petals. Hank is a creditor of Petals, but is not a partner in the partnership.

Point Two (25%)


As the transferee of a legally transferable interest in the partnership, Ivan is legally entitled to
receive distributions that Gina would otherwise have received.

As a partner in the Petals partnership (see Point One), Gina has a transferable interest in
the profits and losses of the partnership and in her right to receive distributions. UPA (1997)
§ 502; UPA (1914) § 26. The transfer of that interest is permissible and creates in the transferee a
right to receive distributions to which the transferor would otherwise be entitled. UPA (1997)
§ 503(b)(1); UPA (1914) § 27(1). Here, Gina transferred her right to receive a share of the net
profits of Petals to Ivan when she assigned all of her interest in Petals to him. Therefore, Ivan is
legally entitled to receive Gina’s share of the net profits of Petals.
[NOTE: An applicant might add that a partnership need not give effect to a transferee’s
rights until it has notice of the transfer. UPA (1997) § 503(e). Here, the partnership had such
notice because Gina gave Fran a copy of the letter effecting the transfer. An applicant might also
note that the transfer of a partner’s right to profits and losses and the right to receive distributions
does not effect a termination or dissolution of the partnership.]

Point Three (25%)


Ivan is not entitled to inspect the books and records of Petals because he is not a partner in
Petals.

As a partner in the Petals partnership, Gina has a transferable interest in the partnership
(see Point Two). That interest, however, is limited. For example, a transfer of the partnership
interest does not make the transferee a partner unless the other partner or partners consent to
making the transferee a partner. On our facts, such consent was not given because Fran said, “I
don’t want anything to do with Ivan.” Therefore, Ivan did not become a partner and is not
entitled to access information concerning partnership transactions nor to inspect or copy the
partnership books or records. UPA § 503(a)(3); UPA (1914) § 27(1).

Point Four (15%)


The delivery truck is partnership property, and Fran is not entitled to use it on Sundays to take
her children to their soccer games.

Property is partnership property if it is acquired in the name of the partnership. UPA


(1997) § 204(a)(1); UPA (1914) § 8(1). Partnership property is property of the partnership and
not of the partners individually. UPA (1997) § 203; UPA (1914) § 25(1). Here, the delivery truck
was purchased in the partnership’s name and, therefore, is partnership property and not Fran’s
individual property. The facts do not suggest that the partners agreed that Fran could use the
delivery truck for her own purposes on Sundays, and, absent such an agreement, Fran is not
entitled to use the truck to take her children to their soccer games even if the partnership is not
using the truck on that day.

[NOTE: Under § 27 of the UPA (1914) and § 503 of the UPA (1997), the transfer of all or any
part of a partner’s interest is not a dissolution of the partnership. Thus, Gina’s gift to her son Ivan
of her interest in the partnership does not dissolve the partnership.]
16
Contracts and Negotiable Instruments Analysis

(Contracts I.A., IV.D., V.;

Negotiable Instruments [Commercial Paper] I.; II.A. & B.; III.A. & B.; IV.B. & D.; V.G.)

Legal Problems: (1) Is a seller bound by statements about goods made by him in
conjunction with their sale?

(2) Under what circumstances may a buyer revoke acceptance of


goods that do not conform to the contract?

(3) What are the rights of a buyer who has revoked acceptance of
goods?

(4) What are the rights of a holder of a dishonored check?

DISCUSSION

Summary

The transaction between Buyer and Seller is a contract for the sale of goods governed by
Article 2 of the Uniform Commercial Code, as well as by common law doctrines (e.g., involving
fraud and misrepresentation) that supplement the Code. Under both UCC principles and common
law principles, Seller’s untrue statements about the brakes and clutch provide a basis for Buyer
to avoid the contract, recover the purchase price, and recover any other allowable damages for
losses that he may have suffered as a result of the breach.
The false statements about the brakes and the clutch were misrepresentations under
common law principles. They also constituted express warranties under the UCC, which Seller
breached because the statements were untrue. Under common law rules, Seller’s
misrepresentations provide a basis for Buyer to avoid the contract because the misrepresentations
were fraudulent and material, they induced Buyer to complete the car purchase, and Buyer was
justified in relying on them. Similarly, under the UCC, Buyer was entitled to revoke his
acceptance of the car because acceptance was induced by Seller’s false assurances. If Buyer
avoids or rescinds the contract under common law principles, or revokes his acceptance under
UCC rules, Buyer is entitled to return of any payments made plus any damages for breach of
contract.
Checkco, as the holder of the check, can recover the amount of the check from Buyer (as
drawer) or Seller (as indorser). Checkco is a holder in due course, and Buyer will have to pay the
amount of the check even though the car did not conform to the contract.

Point One (20%)


Seller’s statements about the brakes and clutch constituted express warranties that became part of
the contract under the Uniform Commercial Code. They were also misrepresentations that could
provide Buyer with a basis to avoid the contract under common law principles.

17
Contracts and Negotiable Instruments Analysis

This is a contract for the sale of goods and as such is governed by Article 2 of the UCC.
In addition, basic principles relating to fraud and misrepresentation can be applied to supplement
the rules in the Code. See UCC § 1-103.
Buyer and Seller clearly entered into a contract for the sale of the car. There was offer
and acceptance, and the basic terms of the bargain were reflected in a writing.
Under the UCC, affirmations of fact relating to the goods that are part of the basis of the
bargain create express warranties that the goods will conform to those affirmations and
descriptions. UCC § 2-313(1). Seller’s statements that the brakes and clutch had been replaced
were clearly such affirmations of fact and, thus, constituted express warranties. The more general
statements that the car was in “tip-top shape” and was “good for another 100,000 miles,”
however, may be “an affirmation merely of the value of the goods or a statement purporting to be
merely the seller’s opinion or commendation of the goods” and, therefore, may not constitute
express warranties. UCC § 2-313(2).
[NOTE: Although the warranties are not included in the writing the parties signed, UCC
§ 2-202 (Article 2’s parol evidence rule) allows that writing to be supplemented by “consistent
additional terms” unless the court finds that the writing was intended to be a “complete and
exclusive” statement of the terms. There are no facts in the problem to suggest that the parties
intended their writing to be the exclusive statement of their terms, and the express warranties
made by Seller were not inconsistent with anything in the writing.]
Under common law principles, a misrepresentation is any “assertion that is not in accord
with the facts,” Restatement (Second) of Contracts § 159, and a misrepresentation is “material” if
it is likely to induce assent to the contract, id. § 162. Here, Seller’s misrepresentations about the
brakes and the clutch were clearly not in accord with the facts, and were also the sort of claims
that would contribute directly and substantially to a buyer’s decision to purchase the car.
Furthermore, there is strong reason to believe that Seller’s statements were fraudulent, as Seller
very likely knew that the brakes and clutch had not been repaired within the last six months
(although the facts are not completely clear on this point). Seller will be bound by his fraudulent
or material misrepresentations. See RESTATEMENT (SECOND) OF CONTRACTS § 164 (buyer may
void a contract if assent is induced by a fraudulent or material misrepresentation on which the
buyer is justified in relying).

Point Two (20%)


Buyer accepted the car, but was entitled to revoke his acceptance under UCC rules because the
car’s nonconformity substantially impaired its value to him and because the car was accepted
without discovery of the nonconformity and acceptance was induced by Seller’s false assurances.
Under the common law doctrine of misrepresentation, Buyer may avoid the contract because his
assent to it was induced by Seller’s misrepresentations.

Buyer took the car and drove it for a week. Since this time span was well beyond a
reasonable period in which to inspect the car, Buyer’s failure to reject the car means that he
accepted it. UCC § 2-606. Having accepted the goods, Buyer can no longer reject them. UCC
§ 2-607(2). However, under proper circumstances, a buyer can revoke acceptance of goods. See
UCC § 2-608. In particular, a buyer may revoke acceptance if (i) the goods do not conform to the
contract, (ii) the nonconformity “substantially impairs” the value of the goods to the buyer, and
(iii) the buyer accepted the goods either on the reasonable assumption that a nonconformity
would be cured or without discovery of a nonconformity if acceptance was induced by either
difficulty of discovery before acceptance or the seller’s assurances; revocation must occur within

18
Contracts and Negotiable Instruments Analysis

a reasonable time after the buyer discovers or should have discovered the nonconformity and
before any change in condition of the goods not caused by the nonconformity. Id.
In this case, all conditions for revocation are met. First, there is no doubt that the goods
do not conform to the contract. The car was warranted to have new brakes and a new clutch. (See
Point One.) Second, it is obvious that those nonconformities substantially impair the value of the
car to Buyer. Third, Buyer accepted the car without discovery of the nonconformities, and this
acceptance was induced by Seller’s false assurances. Moreover, Buyer communicated the
revocation to Seller immediately upon learning of the grounds for it, and there was no change in
the condition of the car that was unrelated to its nonconformity. Therefore, Buyer’s revocation is
effective. See UCC § 2-608(2).
Under the common law doctrine of misrepresentation, a buyer can avoid (rescind) a
contract if his assent was induced by fraudulent or material misrepresentations and he was
justified in relying on those misrepresentations. RESTATEMENT (SECOND) OF CONTRACTS § 164.
Those conditions are met here. Although the facts do not explicitly say so, Buyer asked
specifically about the car’s condition and even contemplated taking it to a mechanic. Seller’s
specific statements about the recent work done on the brakes and clutch (relatively expensive
items to repair) probably were a significant factor in Buyer’s decision to assent to the contract
without inspection. See id. § 167, cmt. a (“It is enough that the manifestation substantially
contributed to his decision to make the contract.”) Buyer was justified in relying on Seller’s
statements about the car’s condition, as Seller was in possession of the car, was offering it for
sale, and represented that he knew its condition. Under these circumstances, Buyer can avoid the
contract as a result of Seller’s misrepresentations.

Point Three (20%)


Because Buyer’s revocation of acceptance is effective, Buyer may cancel the contract, recover
the purchase price that has been paid, and recover any damages. If Buyer chooses to rescind or
avoid the contract under common law principles, Buyer is entitled to restitution of the purchase
price that has been paid, and may also recover any damages.

A buyer who has justifiably revoked acceptance of goods is entitled to recover so much
of the price as has been paid. See UCC § 2-711. Similarly, a buyer who rescinds a contract due to
a seller’s misrepresentation is entitled to restitution of any portion of the purchase price that has
been paid. RESTATEMENT (SECOND) OF CONTRACTS § 376. Thus, Buyer is no longer obligated to
pay for the car and can recover the purchase price that has been paid (the $4,000 in cash and the
$1,000 amount of the check).
Moreover, Buyer is also entitled to damages under UCC § 2-712 or § 2-713. Under UCC
§ 2-713, Buyer is entitled to the difference between the market price of a car conforming to the
contract and the contract price, plus incidental and consequential damages. Because, according to
the mechanic, the market price of a car conforming to the contract was equal to the contract
price, Buyer is entitled only to any incidental and consequential damages under UCC § 2-713.
Alternatively, under UCC § 2-712, Buyer may “cover” by buying a car in substitution for the car
bought from Seller. Such a covering purchase must be a commercially reasonable substitute as
opposed to an upgrade to a better car. If Buyer makes a covering purchase, Buyer may recover
the excess, if any, of the cover price over the contract price of $5,000, plus incidental and
consequential damages. UCC § 2-712(2). Pursuant to UCC § 2-721, these additional remedies
are also available to a buyer who chooses to rescind the contract based on a claim of material
misrepresentation or fraud.

19
Contracts and Negotiable Instruments Analysis

Point Four(a) (20%)


Checkco, as a holder of the check, can assert a claim for the amount of the check against either
Seller or Buyer.

The facts state that Buyer paid $1,000 of the purchase price of the car with a check drawn
on First Bank. A check is a negotiable instrument governed by Article 3 of the Uniform
Commercial Code. UCC § 3-104. Buyer was the “drawer” of the check. See UCC § 3-103(a)(3).
Seller was an “indorser.” See UCC § 3-204(b).
If a check is dishonored, the drawer (Buyer, in this case) has an obligation “to pay [it] . . .
according to its terms at the time it was issued” to a “person entitled to enforce” it. UCC § 3­
414(b). Similarly, if a check is dishonored, an indorser (Seller, in this case) has an obligation to
pay the amount of the check to a person entitled to enforce it. See UCC § 3-415(a). The refusal
of First Bank to pay the check when it was presented by Checkco constituted dishonor. UCC § 3­
502(b)(2).
A “holder” of a check is a person entitled to enforce it. UCC § 3-301. When Seller
indorsed the check in blank (i.e., by signing only his name), the check became payable to the
bearer. UCC § 3-205(b). When Seller handed this “bearer instrument” to Checkco, Checkco
became a holder of the check because it was in possession of an instrument that was payable to
the bearer. UCC § 1-201(b)(21)(A). Thus, under UCC §§ 3-414 and 3-415, Buyer and Seller are
liable to Checkco for the amount of the check.
[NOTE: Seller is liable on the check because Seller indorsed it. Some applicants might
also note that Seller could be liable to Checkco for breach of a transfer warranty because, at the
time Seller transferred the check to Checkco, Seller would have been aware that Buyer had a
defense to payment of the instrument. UCC § 3-416(a). An examinee might be given credit for a
correct analysis of this point, but the more direct route to Seller’s liability is that Seller has an
indorser’s obligation to pay the holder of a dishonored instrument. UCC § 3-415(a).]

Point Four(b) (20%)


Checkco is a holder in due course and will not be subject to defenses that may be asserted by
either Seller or Buyer.

Seller has no defenses to his obligation under UCC § 3-415, so Checkco may recover
from Seller. Buyer, however, might claim that he has a defense to that obligation or a “claim in
recoupment” arising from Seller’s breach of contract in the transaction that gave rise to the
check. If Checkco qualifies as a holder in due course, Buyer will not be able to raise any claims
in recoupment and will be able to raise only the four “real” defenses listed in UCC § 3-305(a)(1).
None of those defenses (infancy, illegality of the transaction, fraud that induced the obligor to
sign the instrument without knowledge of or reasonable opportunity to learn its terms, and
discharge in insolvency proceedings) is present. Thus, if Checkco is a holder in due course, it
will be able to recover from Buyer despite the fact that Seller breached the contract giving rise to
the check.
A holder in due course is a holder of an instrument who takes an instrument that is not
questionable on its face for value, in good faith, and without notice of, inter alia, any defenses to
it. UCC § 3-302(a).
Each of these elements has been fulfilled. First, it has already been established that
Checkco is a holder. Second, there are no facts to suggest that the check was questionable on its
face. Third, Checkco took the check for value because it performed its promise of paying $950

20
Contracts and Negotiable Instruments Analysis

for it. See UCC § 3-303(a)(1). The fact that $950 is less than the face amount of the check is
irrelevant. Fourth, Checkco appears to have taken the check in good faith. “Good faith” is
defined to include “honesty in fact and the observance of reasonable commercial standards of
fair dealing.” UCC § 3-103(a)(4). Nothing in these facts suggests a lack of good faith on the part
of Checkco. There is no evidence of dishonesty and no indication that its actions were outside
the bounds of commercial standards of fair dealing. Finally, there are no facts to suggest that
Checkco had notice of any defenses to the check.
Accordingly, Checkco is a holder in due course and may recover from Buyer
notwithstanding the fact that Seller breached the contract giving rise to the check.
[NOTE: Of course, Checkco is entitled to only one recovery.]

21
Decedents’ Estates Analysis

(Decedents’ Estates I.B.; II.A., B., C., F.1. & I.12)

Legal Problems: (1) Did Testator execute a valid will when he did not sign the only
page containing bequests, but all three pages of the will were
physically present at the time he signed the last page and Testator
intended them to be his will?

(2) Was Testator’s handwritten codicil valid?

(3) Did Testator revoke the bequest to Sister by striking out the clause
in his will giving her $20,000?

(4) Does Testator’s gift to “children” include adopted children and


nonmarital children?

(5) May a court reform Testator’s will when Testator’s likely intention
conflicts with the will as drafted?

DISCUSSION

Summary

Under the doctrine of integration, all pages of a will that are together when the last page
is signed and witnessed are deemed to have been validly executed. Thus, Testator’s will is valid.
Testator’s handwritten codicil is invalid because it was neither signed by Testator nor witnessed.
However, although crossing out a portion of a will (here, the bequest to Sister) would normally
revoke that bequest under this state’s law, because of the doctrine of dependent relative
revocation, Testator’s bequest to Sister was not revoked because it appears that Testator struck
out this bequest on the mistaken assumption that the handwritten codicil was valid. In all states, a
gift to children includes adopted and marital children. In most states, a gift to children would
include a nonmarital child if clear and convincing evidence of paternity was available. Although
Testator appears to have intended to benefit only some of his children, Abby and Bruce, the will
gives the residuary estate to all of Testator’s “children,” and most courts do not allow the
admission of extrinsic evidence to alter the plain meaning of a will. Thus, Sister would receive
$20,000 from Testator’s estate and the balance would likely be distributed, in equal shares, to
Abby, Bruce, Carl, and Don, if Don can establish Testator’s paternity.
If the will was not validly executed Testator would have died intestate and his estate
would be distributed only to his heirs. His heirs would be Abby, Bruce, and Carl in all states and,
depending upon the state rules relating to proof of paternity, possibly Don.

Point One (25%)


Because all pages of Testator’s will were present at the same time that Testator signed the last
page of his will and the pages were intended by Testator to be his will, the will is valid.

22
Decedents’ Estates Analysis

Under the relevant state statute, a will, to be valid, must be signed by the testator and
witnessed by two witnesses. Like all wills statutes, the statute applicable here does not require
that the pages of the will be stapled or that the testator and witnesses sign each page.
Under the doctrine of integration, a multi-page will is valid even if only the last page is
executed as long as the proponent of the will can establish that all pages were physically present
and together when the testator and witnesses signed the last page of the will and that each page
was intended by the testator to be part of his will. See THOMPSON ON WILLS § 124 (3d ed. 1947);
2 PAGE ON THE LAW OF WILLS §§ 19.10, 19.11 (2003). Here, all three pages of the will were
present and together when Testator signed the will, and Testator intended the document to be his
will. The proponent of the will may also rely upon a presumption that all pages of the will were
present and together when the last page was signed if the pages, when read together, set out an
orderly dispositional plan. See THOMPSON, supra, § 105; Comment, Wills: Integration and
Incorporation by Reference, 4 BAYLOR L. REV. 211 (1952); Comment, Wills—Integration, 50
MICH. L. REV. 915 (1952). Here, page 1 of the will contained a bequest consistent with
Testator’s instructions to the attorney, and this page was followed by other pages containing
matters commonly included in wills.
In sum, the will was executed properly and is valid.

Point Two (15%)


Testator’s handwritten codicil to the will is invalid because state law requires all testamentary
provisions to be signed by the testator and witnessed by two attesting witnesses.

Under state law, all testamentary provisions, to be valid, must be signed by the testator
and witnessed by two attesting witnesses. Here, Testator attempted to change the bequest to
Sister with a handwritten, unsigned, and unwitnessed writing. This provision is ineffective under
state law.

Point Three (30%)


Notwithstanding the fact that Testator crossed out the bequest to Sister, the bequest is not
revoked. Under the doctrine of dependent relative revocation, a revocation based on a mistaken
assumption of law or fact (here, the validity of the handwritten bequest) is not given effect if it
appears that the testator would not have revoked his will or some portion thereof had he known
the truth.

Testator crossed out the bequest to Sister with the intention of substituting for it a larger
bequest. This alternative bequest was invalid because it was neither witnessed nor signed by
Testator. (See Point Two.)
Under the doctrine of dependent relative revocation, if a testator revokes a will or bequest
based on a mistaken assumption of law or fact, the revocation is ineffective if it appears that the
testator would not have revoked the bequest had he had accurate information. Here, Testator’s
cancellation of the bequest to Sister was motivated by a desire to give Sister more money. Had
Testator known that the alternative larger bequest would be invalid, he would not have wanted to
cancel the lesser bequest to Sister, leaving her with nothing at all. Thus, under the doctrine of
dependent relative revocation, the revocation of the bequest to Sister is ignored and she is
entitled to the $20,000. See WILLIAM M. MCGOVERN & SHELDON F. KURTZ, 242–43 (3d ed.
2004).

23
Decedents’ Estates Analysis

Point Four (25%)


The bequest to “my children” would include Testator’s two biological children born during
marriage and his adopted stepchild. It is unclear whether it would include Testator’s nonmarital
child.

State laws generally provide that the word “children” in a will should be interpreted
consistently with the definition of the word “children” used in determining rights to intestate
succession. See, e.g., UNIF. PROBATE CODE § 2-705.
Biological children born to a married couple and adopted children are included within the
category of children entitled to take through intestate succession. Thus, Abby, Bruce, and Carl
are all included in Testator’s bequest to his “children.” See MCGOVERN & KURTZ, supra, at
100–04.
In all states, a nonmarital child (here, Don) is included within a bequest to children only
if paternity is established under the relevant statute. In most states, paternity for purposes of
intestate succession may be established by evidence of subsequent marriage of the parents, by
acknowledgment by the father, by an adjudication of paternity during the life of the father, or by
other clear and convincing evidence. See JESSE DUKEMINIER & STANLEY M. JOHANSON, WILLS,
TRUSTS, AND ESTATES 115 (6th ed. 2000). It is unclear whether Testator’s paternity of Don was
adjudicated or acknowledged during Testator’s lifetime; it is also unclear whether there is clear
and convincing evidence of Testator’s paternity.
In sum, it is uncertain whether Testator’s nonmarital child, Don, will be included within
the bequest to Testator’s “children.”
[NOTE: If evidence of Testator’s paternity is not currently available, Don might seek an
order requiring decedent’s marital children to submit to blood testing in order to establish
Testator’s paternity; some courts have granted such petitions. See Sudwischer v. Estate of
Hoffpauir, 589 So. 2d 474 (La. 1991). In recent years, some courts have also permitted
exhumation of a decedent’s remains for purposes of establishing paternity. See Brancato v.
Moriscato, 2003 WL 1090596, 34 Conn. L. Rptr. 208 (Super. Ct. 2003). Other courts have
refused to grant such requests. See In re Estate of Bonanno, 745 N.Y.S.2d 813 (Sur. Ct. 2002)
(request for exhumation denied when postmortem DNA analysis of decedent’s blood samples
established that petitioner was not decedent’s son).]

Point Five (5%)


Most jurisdictions disallow reformation of a will to correct a mistake. In those states, the bequest
to “my children” would stand despite Testator’s apparent intention to benefit only his biological
children, Abby and Bruce.

The fact that Testator identified only Abby and Bruce as his children when talking with
his lawyer is unlikely to result in an interpretation of the will limiting the word “children” to
Abby and Bruce. Although the “controlling consideration in determining the meaning of a
donative document is the donor’s intention” (RESTATEMENT (THIRD) OF PROPERTY: WILLS AND
OTHER DONATIVE TRANSFERS § 10.1 (2003)), a majority of jurisdictions bar the admission of
extrinsic evidence to vary the literal meaning of the words used in a will and refuse to allow
reformation of a will to correct a mistake. “Hypothetical or imaginary mistakes of testators
cannot be corrected. . . . The only means for ascertaining the intent of the testator are the words
written and the acts done by him.” Sanderson v. Norcross, 136 N.E. 170, 172 (Mass. 1922).

24
Decedents’ Estates Analysis

Under this formalist approach, Testator’s attorney could not testify that Testator intended to
leave his estate to only Abby and Bruce, based on their conversations.
In recent years, a few states have begun to move away from the formalist approach and
toward the more liberal approach adopted in the Restatement (Third) of Property. Under the
Restatement, a court may reform even an unambiguous donative document based on clear and
convincing evidence of (a) a mistake of fact or law or (b) the donor’s intention. See
RESTATEMENT (THIRD) OF PROPERTY: WILLS AND OTHER DONATIVE TRANSFERS § 12.1; Estate
of Herceg, 747 N.Y.S.2d 901 (Sur. Ct. 2002) (reforming will based on drafting attorney’s
testimony that omission of particular beneficiary was due to computer error); UNIF. PROBATE
CODE § 2-805 (2008 amendment). Using this approach, a court would admit the attorney’s
testimony to show that Testator’s bequest to “children” was based on a mistake of law (the
meaning of “children” for purposes of intestate succession) and that Testator intended to benefit
only Abby and Bruce. However, it is unclear whether such testimony would meet a clear and
convincing evidentiary standard given that Testator did not discuss his intentions toward Carl
and Don with the attorney.
In sum, Abby and Bruce will certainly take a share of Testator’s estate. It is highly
probable, but not certain, that Carl will. It is uncertain whether Don will as well, depending on
whether Testator’s nonmarital children are included in the class gift to “children” under state
law. See Point Four.

25
Constitutional Law Analysis

(Constitutional Law IV.F.1. & 2.a.)

Legal Problems: (1) May a municipality prohibit leafleting in a public forum based
on an anti-leafleting rationale?

(2) May a public school deny use of classroom space to a student


religious group when, by making that space available to other
student groups, it has created a limited public forum?

(3) May a speaker be convicted for trespassing on government


property that is not a public forum when the government has an
interest unrelated to speech in preventing the trespass?

DISCUSSION

Summary

For First Amendment purposes, there are three types of forums: the public forum, the
limited public forum, and the nonpublic forum. A public street or sidewalk is a public forum. In a
public forum, the state may not regulate speech based on its content, nor may it penalize speech
based on an anti-leafleting rationale. Thus, Homestead may not fine Chapter under the anti-
leafleting ordinance.
When a governmental entity deliberately opens government property to the public for
meetings or other forms of speech, it creates a limited public forum subject to the same First
Amendment requirements as a traditional public forum. Thus, because High School created a
limited public forum, Principal may not deny Church Club the right to use that forum.
In a nonpublic forum, governmental actors may restrict speech as long as their actions are
reasonable and content-neutral. Principal’s office was a nonpublic forum because it was clearly
off-limits to the public except by invitation, and Principal’s actions were reasonable under the
circumstances. Thus, Father has no First Amendment grounds to vacate his trespass conviction.

Point One (40%)


Homestead cannot enforce its anti-leafleting ordinance against Chapter because a municipality
must allow speech in a traditional public forum even if doing so imposes costs on the
municipality.

The First Amendment, as applied to the states and local governments through
incorporation in the Fourteenth Amendment, applies to Homestead’s enforcement of its anti-
leafleting ordinance, which is a state action.
Although the United States Supreme Court once granted local governments broad powers
to regulate speech in streets and parks, see Davis v. Commonwealth of Massachusetts, 167 U.S.
43 (1897), for the better part of a century, the Court has treated a public street or sidewalk as a
First Amendment “public forum.” In a public forum, government may regulate speech only if

26
Constitutional Law Analysis

certain conditions are met. If the regulation is not content-neutral, it must meet the requirements
of strict scrutiny. If the regulation is content-neutral, it must meet the requirements of
intermediate scrutiny; to pass muster under this standard, the regulation must impose only
reasonable restrictions on the time, place, or manner of speech that are narrowly tailored to serve
a significant governmental interest, and it must leave open ample alternative opportunities to
engage in speech. See Ward v. Rock against Racism, 491 U.S. 781 (1989); ERWIN
CHEMERINSKY, CONSTITUTIONAL LAW: PRINCIPLES AND POLICIES 1127 (3d ed. 2006).
Here, Homestead’s regulation of speech under its anti-leafleting ordinance is content-
neutral; the ordinance forbids one means of communication, leafleting, without regard to the
content of the speech. Thus, Homestead must show that its ordinance passes intermediate rather
than strict scrutiny.
The Homestead ordinance is unlikely to pass muster under intermediate scrutiny because
Homestead has not left open an alternative channel for the prohibited speech. Instead of
regulating the time, place, and manner of leafleting, Homestead has banned it entirely.
Particularly where a form of speech is inexpensive and available to all, a governmental entity
may not prohibit it altogether. See City of Ladue v. Gilleo, 512 U.S. 43, 56 (1994).
The fact that the prohibited speech imposes costs upon the governmental entity does not
alter this result. In the seminal case of Schneider v. State of New Jersey, Town of Irvington, 308
U.S. 147 (1939), the Supreme Court struck down an anti-leafleting ordinance much like the
Homestead ordinance. In so doing, the Court held that
“the purpose to keep the streets clean and of good appearance is insufficient to justify an
ordinance which prohibits a person rightfully on a public street from handing literature to
one willing to receive it. Any burden imposed upon the city authorities in cleaning and
caring for the streets as an indirect consequence of such distribution results from the
constitutional protection of the freedom of speech and press.” Id. at 162.
Thus, Homestead may not fine Chapter for leafleting.

Point Two (35%)


A public school that makes classroom space available to student groups has created a limited
public forum and may not discriminate as to use of that forum based on the content of speech.
Thus High School cannot deny use of a classroom after school hours to Church Club.

A “limited public forum” (also known as a “designated public forum”) is created when a
governmental entity could close a location to speech but instead opens it to speech. All the rules
applicable to a traditional public forum apply to a limited public forum. Thus, under the First
Amendment, as applied to the states and local governments through incorporation in the
Fourteenth Amendment, the government may not discriminate among speakers based upon the
content of their speech. See CHEMERINSKY, supra, at 1137.
There is no exception to the requirement of content neutrality when religious speech is at
issue. The Supreme Court has repeatedly held that, in a public forum, religious speech stands on
an equal footing with nonreligious speech, and content-neutral access rules do not violate the
Establishment Clause. Many of the cases have involved student or community groups that
wanted to use, for religious speech, limited public forums created by public schools. See, e.g.,
Widmar v. Vincent, 454 U.S. 263 (1981); Lamb’s Chapel v. Center Moriches Union Free School
Dist., 508 U.S. 384 (1993); Good News Club v. Milford Central School, 533 U.S. 98 (2001). In
Good News, the case most like this one, the Supreme Court held that, when a school opens its
classrooms for use by student groups, it creates a limited public forum and may not deny access

27
Constitutional Law Analysis

to a student religious club that intends to use the classroom for prayer based on the religious
content of that speech. The Court also held that a school that opens its facilities to religious
speech under a content-neutral policy does not violate the Establishment Clause under the
standard established in Lemon v. Kurtzman, 403 U.S. 642 (1971). See Good News, 533 U.S. at
103–04.
Thus, High School may not deny Church Club use of its classrooms for after-school
meetings.

Point Three (25%)


The First Amendment does not shield Father from a trespass conviction. First, government can
regulate conduct that is not imbued with communicative value. Second, even if the conduct has
communicative value, the state can regulate such conduct in a nonpublic forum if the state’s
interest is important or substantial, and is unrelated to the suppression of expression, and if the
regulation is no greater than necessary to serve the state’s interest. Here, Principal’s office was a
nonpublic forum, Father’s act of trespass was not communicative, and High School had an
important and substantial interest in preventing trespassers from entering Principal’s office.

Father was convicted for his physical entry into Principal’s office, not for what he said
while there. The act of marching into Principal’s office—which is the act for which Father was
convicted—is not conduct imbued with communicative value and does not implicate the First
Amendment. Thus, Father’s trespass conviction will likely survive a First Amendment challenge.
Even if Father’s conduct is viewed as conduct imbued with communicative value because
he entered Principal’s office to express an opinion, the conviction would be upheld under United
States v. O’Brien, 391 U.S. 367 (1968). The test in that case is whether a law that regulates
speech-related conduct serves an interest within the government’s regulatory powers that is
important or substantial and that is both unrelated to the suppression of expression and no greater
than necessary to serve the state’s interest. Here, Principal’s office was clearly not a public
forum. The door was closed, a sign announced that there was “no admittance without an
appointment,” and Principal was meeting with another parent at the time that Father marched in.
High School clearly has a legitimate and important interest in prohibiting trespassers from
entering Principal’s office where private conversations about individual students or official
school business may be taking place. High School’s interest is not related to suppression of
expression. Moreover, the means of protecting that interest (prohibiting admittance without
appointment) are closely related to advancement of High School’s interest.

28

Real Property Analysis

(Real Property I.A.1. & 3., B., B.4.)

Legal Problems: (1) What interest did School acquire in Blackacre and what interest did
Owner retain?

(2) Did School breach the condition or limitation in the deed from
Owner when it stopped using the building erected on Blackacre to
teach students and began using it for administrative purposes?

(3) If Owner retained a future interest in Blackacre, did the statute of


limitations run on that interest?

(4) What interests, if any, do Ann and Mary have in Blackacre?

DISCUSSION

Summary

Owner conveyed either a fee simple determinable or a fee simple on condition


subsequent to School. Because the Owner-to-School deed used language that did not conform to
the classic formulations for creating either of these interests, it is unclear which interest School
acquired. However, in the case of an ambiguous conveyance, there is a presumption in favor of
the fee simple on condition subsequent.
If School has a fee simple on condition subsequent, either (1) Owner retained nothing to
pass to Daughter because the Owner-to-School deed did not expressly specify that Owner
retained a power of termination, or (2) Owner retained an implied power of termination which
passed to Daughter and then through Daughter’s estate to Ann and possibly, upon Bill’s death, to
Mary as well.
If School has a fee simple determinable, then Owner’s retained possibility of reverter
passed through Daughter’s estate at least to Ann and possibly to Mary.
Any cause of action for breach of the condition or limitation (depending on the interest
created in the School) accrued only three years ago and, thus, the statute of limitations on that
action has not run.
Of course, whether School’s interest has been lost depends upon whether using Blackacre
for administrative purposes violates the requirement of using the property to teach children aged
5 to 13, which it probably does.

Point One (45%)


The terms of the conveyance to School were ambiguous. In construing an ambiguous instrument,
courts typically adopt a preference for the fee simple on condition subsequent. If so construed, it
is unclear what interest, if any, Owner retained. It is either no interest or a right of entry for
condition broken (also called a “power of termination”). If, on the other hand, School acquired a
fee simple determinable in Blackacre, then Owner retained a possibility of reverter.

29
Real Property Analysis

If the Owner-to-School deed conveyed a fee simple determinable to School, then Owner
retained a possibility of reverter, which became possessory immediately upon the happening of
the event designated in the instrument. JOSEPH W. SINGER, INTRODUCTION TO PROPERTY 309 (2d
ed. 2005). If, instead, the Owner-to-School deed conveyed a fee simple on condition subsequent,
Owner may or may not have a power of termination or right of entry for condition broken,
depending on whether the court would be willing to imply a forfeiture provision when none was
expressly set forth in the deed. See RICHARD POWELL, REAL PROPERTY § 1:13.06[3] (Patrick J.
Rohan ed., 1990); THOMPSON ON REAL PROPERTY § 17.01(d) (David A. Thomas ed., 1994)
Here, Owner conveyed Blackacre to School “if School uses Blackacre only to teach
children aged 5 to 13.” The language of the deed does not conform to the paradigms for creating
either a fee simple determinable or a fee simple on condition subsequent. To create a fee simple
determinable, the typical formulation would be “to School, so long as it uses Blackacre only to
teach children aged 5 to 13.” To create a fee simple on condition subsequent, the typical
formulation would be “to School, but if School does not use Blackacre only to teach children 5 to
13, then the grantor may reenter and reclaim Blackacre.” See SHELDON F. KURTZ, MOYNIHAN’S
INTRODUCTION TO THE LAW OF REAL PROPERTY 44–48 (4th ed. 2005).
In this case, the deed language is ambiguous. The word “if” in the deed expresses the
language of condition and suggests an intent to create a fee simple on condition subsequent, but
the deed’s lack of an express, retained power of termination suggests the intention to instead
create a fee simple determinable. On the other hand, the deed’s failure to use the typical
formulation for creating a fee simple determinable, coupled with the absence of any language
suggesting that Owner intended to retain a possibility of reverter (e.g., “Blackacre shall revert”),
suggests that Owner did not intend to convey a fee simple determinable, but instead a fee simple
on condition subsequent.
Where the terms of a conveyance are ambiguous, courts construe the instrument to
effectuate the grantor’s intentions. In construing an ambiguous instrument, courts typically adopt
a preference for the fee simple on condition subsequent. See id. at 48; WILLIAM B. STOEBUCK &
DALE A. WHITMAN, THE LAW OF PROPERTY 44 (3d ed. 2000).
Here, it doesn’t matter whether the court finds that the Owner-to-School deed created a
fee simple determinable or fee simple on condition subsequent if, in the latter case, it also implies
a forfeiture provision. Either construction would allow Owner to regain possession of Blackacre
if School ceased using Blackacre only to teach children aged 5 to 13. The only practical
difference that flows from these two constructions is that the fee simple on condition subsequent
would require Owner (or his successors) to first make a demand, and the fee simple determinable
would not because it becomes possessory immediately upon the happening of the limitation. In
either case, the holder might have to sue the School to enforce her rights if Blackacre is not
vacated voluntarily by School.
If, however, the court construes the deed as creating a fee simple on condition subsequent
and does not imply a power of termination, then School has what amounts to a fee simple
absolute because no one has the power of termination or right of entry for condition broken. In
this case, School’s decision to discontinue use of Blackacre for teaching children aged 5 to 13
has no effect on School’s interest in Blackacre.
[NOTE: Applicants who conclude that School has a fee simple on condition subsequent
may fail to realize that a right of entry must be expressly reserved in the deed and, thus, fail to
discuss a court’s reluctance to find a forfeiture.]

30
Real Property Analysis

[NOTE: An applicant’s conclusion on how the deed should be construed is much less
important than demonstrating an understanding of the consequences that flow from the various
constructions.]

Point Two (10%)


School has likely ceased using Blackacre “only to teach children aged 5 to 13.” If Owner is
treated as having either a possibility of reverter or a right of entry for condition broken (power of
termination), then School’s interest can come to an end and, subject to Point Four, Blackacre
passes to the successors of Owner’s estate. Otherwise, School continues to own Blackacre.

There is a factual question as to whether ceasing to only teach children aged 5 to 13 in the
building on Blackacre and then using that building for administrative purposes is inconsistent
with language in the deed from Owner to School. See Mahrenholz v. County Bd. of School
Trustees of Lawrence County, 417 N.E.2d 138 (Ill. Ct. App. 1981). Here again applicants can
make an argument either way, but the stronger argument appears to be that the terminating event
occurred. No teaching appears to be going on in the building; rather, it is being used for
administrative purposes, and that use seems to be beyond the intended purpose set forth in the
Owner-School deed.

Point Three (10%)


If Owner’s successors have an interest in Blackacre, the statute of limitations has not run on that
interest.

Whether Owner conveyed to School a fee simple determinable or a fee simple on


condition subsequent, the state statute of limitations applicable to actions for possession permits
actions within “10 years after the cause of action accrues.”
Because School ceased using Blackacre for the permitted purposes only three years ago,
the period in which an action for possession may be brought has not yet expired.

Point Four (35%)


Ann has an interest in Blackacre if Owner retained either a possibility of reverter or a power of
termination. In some jurisdictions, Mary (Bill’s widow) would also have such an interest. If
Owner retained only a power of termination, Ann will have to assert that right by exercising the
power because possession will not automatically inure to her as it would have if Owner had
retained a possibility of reverter.

If Owner retained a future interest in Blackacre, that interest passed to Daughter upon
Owner’s death because state law provides that such an interest is devisable. Upon Daughter’s
death, Daughter’s devisable interest passed to Husband for life, with the remainder to “my
surviving children.”
Although Daughter was survived by two children, one of them, Bill, predeceased
Husband. According to the Restatement of Property, which expresses what appears to be the
current majority view, a survivorship contingency like that contained in Daughter’s will applies
at the termination of the interests that precede distribution of the remainder. See RESTATEMENT
OF PROPERTY § 251 (1940). In this case, that point is Husband’s death. Thus, under the
Restatement approach, Mary has no interest in Blackacre because Bill, having predeceased
Husband, had no interest in Blackacre to devise to Mary. This approach is typically justified on

31
Real Property Analysis

the ground that a testator would want an interest in property to pass only to children who could
take possession.
There is another view that interprets a survivorship contingency to require surviving only
the testator (here, Daughter) and not the life tenant (here, Husband). This view is typically
justified by a preference for early vesting of estates. If it applies, then Bill’s interest passed to
Mary. See 3 POWELL ON REAL PROPERTY ¶ 27.03[2] (Michael Allen Wolf ed., LexisNexis
Matthew Bender 2008).
If Owner bequeathed a possibility of reverter in Blackacre, instead of a power of
termination, then Ann (and Mary, in a state that follows the non-Restatement view) is entitled to
immediate possession of Blackacre. If Owner retained and bequeathed only a power of
termination, then Ann’s (and, in a non-Restatement jurisdiction, Mary’s) interest would not
become possessory until and unless the power of termination was exercised.

32
Family Law Analysis

(Family Law I.F.; III.D., E.)

Legal Problems: (1) Under what circumstances is a premarital agreement enforceable?

(2) What property is subject to division at divorce?

(3) When is an alimony award appropriate?

DISCUSSION

Summary

Although state rules governing the enforceability of a premarital agreement vary


considerably, a premarital agreement will generally be enforced unless the party against whom
enforcement is sought can show involuntariness, inadequate disclosure by the other spouse, or
unconscionability. Although Husband cannot show involuntariness or inadequate disclosure, it is
possible that he can show unconscionability if this jurisdiction evaluates unconscionability at the
time enforcement of the agreement is sought. If the agreement is unenforceable, in most states
Husband’s inherited real estate would not be divisible at divorce; the marital home titled in
Wife’s name would be, and Wife’s pension would be partly marital and partly separate. In
considering whether to award alimony, courts consider a variety of factors, including the parties’
ages, length of the marriage, contributions by both parties, and the parties’ physical and mental
health. Here, it is unclear whether a court would award Husband alimony.

Point One (50%)


In determining whether a premarital agreement governing property distribution at divorce is
enforceable, courts consider whether the agreement was voluntarily made, whether it is
unconscionable, and whether disclosure of assets and obligations was made. The weight assigned
to these factors varies from state to state. Here, Husband may have a basis for challenging the
validity of the premarital agreement in a state that allows a court to strike down an agreement
based on unconscionability at the time enforcement is sought.

Although courts were once hostile to premarital agreements, today all states permit
spouses to contract premaritally with respect to rights and obligations in property. In all states,
the enforceability of such agreements turns on three factors: voluntariness, unconscionability,
and disclosure. How courts apply these factors varies significantly from one state to the next. In
many states, an agreement is unenforceable if the party against whom enforcement is sought
succeeds in showing any one of the three factors: involuntariness, unconscionability, or
inadequate disclosure.
Under the Uniform Premarital Agreement Act (UPAA), which has been adopted in 27
jurisdictions, the party against whom enforcement is sought must prove (1) involuntariness or (2)
that “the agreement was unconscionable when it was executed” and that he or she did not receive
or waive “fair and reasonable” disclosure and “did not have, or reasonably could not have had,

33
Family Law Analysis

an adequate knowledge” of the other’s assets and obligations. UNIF. PREMARITAL AGREEMENT
ACT § 6(a). Thus, under the UPAA, a court may not refuse to enforce a premarital agreement
based on unconscionability unless the court also finds lack of adequate disclosure or knowledge.
Under the UPAA, Husband and Wife’s premarital agreement would be enforceable. The
facts show that Husband and Wife both disclosed their assets, and there is no evidence that
Husband signed the agreement involuntarily. Although lack of independent legal advice is a
factor in determining whether a party entered into an agreement voluntarily, Husband had ample
opportunity to obtain legal counsel and decided against it. Husband also had ample time to
review the agreement before signing it, and, perhaps most importantly, he proposed the
agreement. See ROBERT E. OLIPHANT & NANCY VER STEEGH, FAMILY LAW §§ 14.11, 14.13 (2d
ed. 2007); Hengel v. Hengel, 365 N.W.2d 16 (Wis. Ct. App. 1985) (agreement upheld when wife
received proposed agreement two weeks before wedding and had it reviewed by legal counsel).
In states that have not adopted the UPAA, courts employ a range of standards. In most of
these states, a court may invalidate a premarital agreement on grounds of unconscionability even
if both parties have fully disclosed their assets. In evaluating unconscionability, some states
require courts to determine whether the agreement was unconscionable when signed; others
require a determination of unconscionability at the time of divorce; yet other states permit courts
to invalidate an agreement based on unconscionability at either point in time or to invalidate an
agreement simply because it is unfair. And some states specifically disallow spousal support
waivers or apply special standards to such waivers. See HARRY D. KRAUSE, ET AL., FAMILY LAW:
CASES, COMMENTS AND QUESTIONS 208 (6th ed. 2007).
Here, the agreement initially benefited Husband by ensuring payment of his debts and
protecting him against his own imprudent speculation; if unconscionability is analyzed at the
time the agreement was signed, the court will likely find it enforceable. At the time of divorce,
however, the agreement is arguably unconscionable in that it would grant Wife not only all
marital assets, but also Husband’s inherited real estate, an asset that, in most states, would be
Husband’s separate property. (See Point Two.) Moreover, Husband has suffered a serious injury
and might succeed in establishing that he is incapable of self-support. Thus, in a state that
permits invalidation of an agreement based on unconscionability at the time of divorce, a court
could find that the premarital agreement between Husband and Wife is unconscionable and
unenforceable. In a state that disallows spousal support waivers, this provision might also be
struck down even if the property division provision were upheld.
[NOTE: A number of states that have adopted the UPAA have varied its enforcement
provisions. For example, although the UPAA provides that “parties to a premarital agreement
may contract with respect to . . . the modification or elimination of spousal support,” UPAA
§ 3(a)(4), some UPAA states have disallowed spousal support waivers (see S.D. CODIFIED LAWS
§ 25-2-18 (2009); Sanford v. Sanford, 694 N.W.2d 283 (S.D. 2005)) or allowed such waivers
only in limited circumstances. See 750 ILL. COMP. STAT. ANN. § 10/7 (West 2010) (a spousal-
support waiver is unenforceable if it causes the waiving spouse undue hardship in light of
circumstances not reasonably foreseeable at the time of signing). Some states permit a court to
invalidate a premarital agreement on grounds of unconscionability even when there has been full
asset disclosure. See, e.g., WIS. STAT. ANN. § 766.58(6) (2009).]

Point Two (30%)


Unless a valid premarital agreement specifies otherwise, in most states, property acquired during
the marriage through spousal effort is divisible at divorce, but property acquired through gift,

34
Family Law Analysis

descent, or devise is not. Thus Husband could be awarded part of Wife’s pension and a share in
the marital home.

In all states, a divorce court may divide marital property without regard to title. In the
majority of states, a divorce court may not divide separate property. In a minority of “hotchpot”
jurisdictions, the court may divide all assets, whenever or however acquired; a few states permit
the division of separate property in special circumstances, such as hardship. See KRAUSE, supra,
at 752, 763–65 (6th ed. 2007).
An asset is marital if it was acquired during the marriage by any means other than gift,
descent, or devise. An asset that is initially separate property may be transformed into marital
property if marital funds or significant efforts by the owner-spouse enhance its value or build
equity during the marriage. See id.
Here, if State A is a majority jurisdiction, Husband’s real estate would be classified as
separate property, the marital home as marital, and Wife’s pension as partly marital and partly
separate. The real estate was inherited, and there is no evidence that its value has been enhanced
by Husband’s efforts or marital funds. The marital home was purchased after marriage with
Wife’s employment income. The value of Wife’s pension reflects both marital employment
income and premarital employment income; a court would thus apportion its value. See J.
THOMAS OLDHAM, DIVORCE, SEPARATION AND THE DISTRIBUTION OF PROPERTY § 7.10[5] (2006)
(describing apportionment principles).
Thus, absent an enforceable premarital agreement, a divorce court could award Husband
a share of Wife’s pension and the marital home. In most states, it could not award Wife a share
of Husband’s real estate.

Point Three (20%)


Even if the premarital agreement is unenforceable, it is unclear whether Husband can obtain
alimony. Husband’s failure to contribute to the marriage weighs against his alimony claim, but,
if Husband can establish that his injury precludes employment, that factor coupled with his age
and the duration of the marriage may support an alimony award.

State A terms spousal support “alimony.”


Rules governing the award of alimony vary from one state to the next, but they almost
invariably require the trial court to consider the parties’ financial resources and needs, their
marital contributions, and the marital duration. Some state statutes also require consideration of
spousal misconduct. See Robert Kirkman Collins, The Theory of Marital Residuals: Applying an
Income Adjustment Calculus to the Enigma of Alimony, 24 HARV. WOMEN’S L.J. 23, 46 (2001)
(describing and categorizing factors in alimony determination); LYNN D. WARDLE & LAURENCE
C. NOLAN, FUNDAMENTAL PRINCIPLES OF FAMILY LAW 718 (2d ed. 2006); AMERICAN LAW
INSTITUTE, PRINCIPLES OF THE LAW OF FAMILY DISSOLUTION: ANALYSIS AND
RECOMMENDATIONS §§ 5.02, 5.03 (2002). Although alimony statutes typically accord substantial
discretion to the trial court, they are invariably gender neutral. See Orr v. Orr, 440 U.S. 268
(1979).
In evaluating need, a court typically considers the standard of living enjoyed by the
parties during the marriage and whether the recipient spouse will be able to achieve that level of
economic self-sufficiency within a reasonable period of time following the divorce; health, the
existence of separate assets, and the share of marital assets that the recipient spouse will receive
may also be taken into account. See OLIPHANT & VER STEEGH, supra, §§ 11.14–11.18.

35
Family Law Analysis

In evaluating a spouse’s contributions to the marriage, a court may consider negative as


well as positive actions, including a spouse’s failure to make economic or noneconomic
contributions and misuse or dissipation of marital funds. Id. § 11.13.
In this case, both Husband and Wife are minimally educated. The 15-year marriage was
relatively lengthy. The marital standard of living has not been high, and throughout the marriage,
Wife has made greatly disproportionate contributions. For most of the marriage, Husband’s
failure to contribute more was unexcused. Although Husband suffered a major injury five years
ago, a court may be skeptical of his claim that he cannot work given Husband’s employment
history, his apparent lack of noneconomic contributions, and his failure to make use of the pilot’s
license he acquired with Wife’s help. Wife does not earn a high salary that would permit her to
pay alimony easily. Finally, because the question assumes that the agreement is invalid, Husband
would, in most states, retain his inherited real estate, which would provide him with an asset that
he could use to defray his expenses.
On the other hand, if Husband succeeds in establishing his disability, a court might find
that his current need outweighs his lack of economic contributions, particularly given the length
of the marriage, Husband’s age, Wife’s earning ability, and the fact that Wife will receive a share
of marital assets.
Thus, on these facts, it is possible, but by no means certain, that Husband will obtain an
alimony award if the premarital agreement is found to be invalid.

36
Federal Civil Procedure Analysis

(Federal Civil Procedure I.A.1., B.)

Legal Problems: (1) May a federal district court exercise personal jurisdiction over a
nonresident defendant that contracted outside the forum state to
provide a loan to a plaintiff who lived in the forum state and that
mailed loan paperwork to the plaintiff in the forum state, including
the incorrectly mailed paperwork that gave rise to the plaintiff's
lawsuit?

(2) Does a federal district court have diversity jurisdiction when the
plaintiff previously was domiciled in the defendant’s state of
citizenship and still returns there about once a month, but has
moved to another state to work and may intend to stay there
permanently?

(3) Does a federal district court have federal-question jurisdiction over


a lawsuit when the plaintiff’s complaint refers to a potentially
relevant federal statute but denies the applicability of the statute?

DISCUSSION

Summary

The State B district court may exercise specific personal jurisdiction over nonresident
defendant Credit Union because Credit Union engaged in contacts with the forum state that
demonstrated purposeful availment and that gave rise to Paul’s claims. The exercise of personal
jurisdiction would not be unfair.
Diversity of citizenship exists if Paul intends to remain in State B, which could be argued
either way. Defendant Credit Union is a citizen of State A, where it is incorporated and has its
principal place of business. Paul’s complaint seeks sufficient monetary damages to satisfy the
amount-in-controversy requirement.
The district court does not have federal-question jurisdiction in this case because Paul’s
complaint does not state a federal claim. It instead seeks to anticipate and rebut a potential
federal defense. Under the well-pleaded complaint rule, federal-question jurisdiction is
appropriate only when a federal claim is raised on the face of the complaint.

Point One (35%)


The United States District Court for the District of State B may exercise personal jurisdiction
over Credit Union because Credit Union’s contacts with the forum state gave rise to the suit and
demonstrate purposeful availment and because the exercise of jurisdiction would not be unfair.

The United States District Court for the District of State B has personal jurisdiction over
Credit Union. Federal district courts may exercise personal jurisdiction to the same extent as the

37
Federal Civil Procedure Analysis

courts of general jurisdiction of the state in which the district court sits. See FED. R. CIV. P.
4(k)(1)(A). State courts of general jurisdiction may exercise personal jurisdiction over
nonresident defendants to the extent authorized by both the state’s long-arm statute and the due
process clause of the 14th Amendment of the U.S. Constitution. In this case, State B’s long-arm
statute authorizes jurisdiction as far as the Constitution allows, so the long-arm analysis is the
same as the constitutional analysis.
The due process clause of the 14th Amendment permits states to assert personal
jurisdiction over nonresident defendants who have established minimum contacts with the state
such that the exercise of personal jurisdiction would not offend traditional notions of fair play
and substantial justice. International Shoe v. Washington, 326 U.S. 310 (1945).
In this case, Paul’s suit arises out of or relates to Credit Union’s contacts with State B.
Paul’s claim is based on a loan agreement which informed Credit Union that Paul was a resident
of State B at the time of the agreement and that loan information should be mailed to Paul in
State B. Credit Union in fact sends information about the loan to Paul in State B, and receives
payments from Paul from State B. Moreover, the lawsuit arises out of one of Credit Union’s
mailings to State B, albeit a mailing to an incorrect address.
Although these contacts probably fall short of the continuous and substantial contacts
required for general jurisdiction, see Helicopteros Nacionales v. Hall, 466 U.S. 408 (1984), they
are probably sufficient for the court to exercise specific jurisdiction over claims against Credit
Union that arise out of its contacts with State B.
A nonresident defendant is subject to specific jurisdiction when its contacts with the
forum state demonstrate purposeful availment of the benefits of the forum state and/or render it
foreseeable that the defendant may be haled into the forum state’s courts. See Hanson v.
Denckla, 357 U.S. 235 (1958). While Credit Union might argue that its contacts with State B
result from Paul’s unilateral act of moving to State B and therefore do not demonstrate
purposeful availment, this argument will fail. Credit Union entered a long-term contract with
Paul knowing that he lived in State B and would be performing his part of the contract (making
loan payments) in State B. Credit Union mailed several payment statements addressed to Paul in
State B. As a result, Credit Union purposefully availed itself of the benefits of State B and could
foresee being haled into court in State B on matters related to the contract with Paul. See Burger
King Corp. v. Rudzewicz, 471 U.S. 462, 475–76 (1985).
Even when a nonresident defendant has established the necessary minimum contacts with
the forum state, the exercise of personal jurisdiction over the defendant may offend due process
if inconsistent with traditional notions of fair play and substantial justice. The defendant must
make a compelling case for fairness considerations to outweigh the existence of minimum
contacts. Id. at 477. Credit Union will not succeed in making such a case, as the fairness
considerations tend to support the exercise of personal jurisdiction by the courts of State B. Paul,
who now lives in State B, has an interest in convenient and effective relief close to home. State B
has an interest in providing its residents with a forum for redress of injuries caused by out-of­
state defendants. And although litigation in State B will impose a burden on Credit Union, which
maintains no branches in State B, that burden is minimized by modern transportation and
communication. See id. at 483–84. As a result, the exercise of personal jurisdiction by the courts
of State B will not offend notions of fairness.

Point Two (35%)


Credit Union is a citizen of State A. As a result, complete diversity will exist if Paul is a
domiciliary of State B. However, it is uncertain whether Paul is a domiciliary of State B or is still

38
Federal Civil Procedure Analysis

a domiciliary of State A. If Paul is a State B domiciliary, then there is diversity jurisdiction;


Paul’s claim satisfies the amount-in-controversy requirement.

28 U.S.C. § 1332 vests the district courts with jurisdiction over cases in which (a) there is
complete diversity of citizenship, meaning no plaintiff shares the state of citizenship of any
defendant, and (b) the amount in controversy exceeds $75,000.
In determining citizenship, § 1332(c) directs that a corporation is a citizen where it is
incorporated and where it has its principal place of business. Hertz Corp. v. Friend, ___ U.S.
___, 130 S.Ct. 1181, 1192 (2010) (“‘[P]rincipal place of business’ is best read as referring to the
place where a corporation’s officers direct, control, and coordinate the corporation’s activities
. . . the place where the corporation maintains its headquarters . . . i.e., the ‘nerve center’”).
Credit Union is incorporated in State A. Moreover, its principal place of business (indeed, its
only place of business) is in State A. Credit Union is a State A citizen for diversity purposes.
An individual U.S. citizen’s citizenship is determined by domicile, which is defined as
the individual’s “true, fixed, and permanent home . . . to which he has the intention of returning
whenever he is absent therefrom.” Mas v. Perry, 489 F.2d 1396, 1399 (5th Cir. 1974) (quotation
omitted). Until recently, Paul’s domicile was clearly State A, where he had lived his entire life.
Six months ago, however, Paul moved to State B, where he now resides most of the time,
although he sometimes returns for short visits to his vacation home in State A.
The question is whether Paul has changed his domicile to State B. To change domicile,
an individual must both take up residence in a new state and intend to remain there. Id. at 1400.
While Paul has taken up residence in State B, it is unclear whether he intends to remain there. He
originally planned to stay in State B for a year and then decide whether he would remain or
return to State A. Consistent with that plan, he took a job as a temporary employee. Recently,
however, Paul decided to obtain more permanent employment, as he has really enjoyed living in
State B. This change to more permanent employment suggests that Paul now intends to remain in
State B on a permanent basis, in which case his domicile has changed to State B. On the other
hand, his purchase of a vacation home in State A may suggest that he does not, in fact, intend to
change his State A domicile.
[NOTE: The issue can be fairly argued either way.]
If Paul is a citizen of State B, complete diversity exists. If Paul remains a citizen of State
A, complete diversity does not exist, as Paul and Credit Union would both be citizens of State A.
To determine whether the amount in controversy exceeds $75,000, courts rely on a
plaintiff’s good-faith allegations unless it appears “to a legal certainty” that the plaintiff cannot
recover the amount alleged. St. Paul Mercury Indemnity Co. v. Red Cab Co., 303 U.S. 283, 289
(1938). Paul’s claim for $240,000 clearly satisfies the amount-in-controversy requirement, and
there are no facts to suggest that it was not made in good faith. Even if the state statute providing
for treble damages is preempted by the federal identity-theft statute, Paul’s $80,000 claim for
actual loss still satisfies the amount-in-controversy requirement.

Point Three (30%)


Paul’s assertion of a defense to the possible applicability of a federal statute does not create
federal-question jurisdiction.

28 U.S.C. § 1331 provides district courts with jurisdiction to hear suits arising under the
laws of the United States. To determine whether such federal-question jurisdiction exists, a court
looks at the allegations necessary for a well-pleaded complaint and asks whether the claims

39
Federal Civil Procedure Analysis

stated by those allegations are created by federal or state law. Claims created by federal, but not
state, law generally fall within the scope of a district court’s federal-question jurisdiction.
Paul’s breach-of-contract claim is created by state law and therefore lies outside the
district court’s original federal-question jurisdiction. Although Paul’s complaint mentions a
federal statute, it does not state any claim that is based on the federal law. Instead, in order to
rebut an anticipated defense, Paul’s complaint asserts that certain provisions of federal law are
unconstitutional.
Pursuant to the well-pleaded complaint rule, there is no federal-question jurisdiction
when federal law is mentioned in the complaint only to deny the applicability of that law.
Federal-question jurisdiction exists “only when the plaintiff’s statement of his own cause of
action shows that it is based upon [federal] laws or [the] Constitution.” Louisville & Nashville
Railroad v. Mottley, 211 U.S. 149, 152 (1908). Here, Paul is asserting a cause of action based
only on state law; the only federal issue in the case is whether Paul’s cause of action is
preempted. Federal-question jurisdiction does not exist under these circumstances.

40
Criminal Law Analysis

(Criminal Law II.A.3; IV.B.1.)

Legal Problems: (1) What are the elements of the crime of larceny by false pretenses?

(2) Did Owner make a false representation of a material present or


past fact that caused Customer to pass title to the property (the
$250) to Owner?

(3) Did Owner act with the mens rea required to be found guilty of
false pretenses?

DISCUSSION

Summary

To be guilty of larceny by false pretenses in most jurisdictions, Owner must have made a
false representation of a material fact that caused Customer to pass title to property to Owner.
Owner must have acted with knowledge of the falsity of the fact and with an intent to defraud.
While Owner did not believe that Star had the best refurbished computers in town, his
claim to the contrary was a false representation of opinion, not fact, and could not support a
conviction for larceny by false pretenses. However, Owner’s statement that the computer
technician installed a new hard drive in each refurbished computer was a false statement of a
material fact because the technician did not install new hard drives. Owner obtained title to
Customer’s property when Customer paid Owner $250 for the computer. The fact that Customer
purchased the computer after being satisfied by Owner’s claims suggests that Customer relied
upon Owner’s false representation of fact in passing title to property.
A court would likely find that Owner knowingly made the false representation about new
hard drives because Owner believed that it was very likely that the computer technician used by
Star did not install new hard drives, and Owner deliberately decided not to question the
technician. Owner also had the requisite intent to defraud because Owner made the false claim
about the new hard drives based upon the belief that it was one of the most important factors that
would lead to a purchase by Customer.

Point One (25%)


To be guilty of larceny by false pretenses, a defendant must knowingly make a false
representation of material fact to a person, with the intent to defraud, and the false representation
must cause the victim to pass title to something of value to the defendant.

The crime of “larceny by false pretenses” or, as it is sometimes called, “obtaining


property by false pretenses” or just “false pretenses,” is a type of theft offense. In most
jurisdictions, the actus reus elements of false pretenses are (i) a false representation of material
fact (ii) that causes another person to transfer title to property (including money) to the
defendant. The mens rea required is knowledge that the representation of fact is false and an

41
Criminal Law Analysis

intent to defraud. See generally JOSHUA DRESSLER, UNDERSTANDING CRIMINAL LAW § 32.10
(5th ed. 2009). See, e.g., MODEL PENAL CODE § 223.3 (theft by deception).
Here, Owner is charged with the crime of false pretenses on the theory that Owner stole
Customer’s money (the $250 that Customer paid for the computer) by lying to Customer in order
to induce Customer to give the money to Owner in exchange for the computer.

Point Two (35%)


The actus reus elements of the crime of false pretenses are met because Owner made a false
representation of a material present or past fact that caused Customer to pass title to something of
value (the $250) to Owner.

As noted above, to be guilty of false pretenses in most jurisdictions, Owner must have
made a false representation of a material present or past fact that caused Customer to pass title to
property to Owner. See State v. Agosta, 787 A.2d 1252, 1254 (Vt. 2001). The false representation
must be one of fact, not opinion. See McMurphy v. State, 455 So. 2d 924, 927 (Ala. Crim. App.
1984). Commercial puffery is not generally considered “false pretenses.” See DRESSLER, supra,
§ 32.10[C][2][a]. While Owner did not believe that Star had the best refurbished computers in
town, his claim to the contrary was a false representation of opinion, not of fact, and could not
support a conviction for larceny by false pretenses. See id.
On the other hand, Owner’s statement that the technician had installed a new hard drive
in each refurbished computer was a false statement of a material fact. See People v. Taurianen,
300 N.W.2d 720, 725 (Mich. Ct. App. 1980) (per curiam) (misrepresentation of fact where
defendants accepted payment for repair work that was not performed). Owner claimed that the
technician had installed a new hard drive in every refurbished computer when no such
installation had occurred.
Through this false representation, Owner succeeded in obtaining title to Customer’s
property when the purchase of the computer was completed and Customer gave Owner $250. See
People v. Long, 294 N.W.2d 197, 200 (Mich. 1980).
It does not matter that Customer may have had additional reasons for purchasing the
computer—reasons that had nothing to do with Owner’s false representation. It is enough that
Owner’s false representation contributed to Customer’s parting with property. See State v.
Handke, 340 P.2d 877, 883 (Kan. 1959). The fact that Customer purchased the computer after
being satisfied by Owner’s claims, including the claim that the computer technician installed a
new hard drive in every refurbished computer, suggests that the false representation contributed
to Customer’s parting with $250, thus establishing the causal link between Owner’s false
representation and the transfer of title to the property. See id.

Point Three (40%)


The mens rea element of the crime of false pretenses is likely met because Owner made the false
representation knowingly and with the intent to defraud Customer.

To be guilty of false pretenses, Owner must have made the false representation
knowingly and with the intent to defraud Customer. See Trevino v. State, 142 P.3d 214, 216
(Wyo. 2006). Most courts find that a defendant acts knowingly and has knowledge of a particular
fact when, inter alia, the defendant is aware of a high probability of the fact’s existence and
deliberately avoids learning the truth. See United States v. Guay, 108 F.3d 545, 551 (4th Cir.
1997); MODEL PENAL CODE § 2.02(7). A court would likely find that Owner knowingly made the

42
Criminal Law Analysis

false representation based upon both Owner’s belief that it was very likely that the computer
technician used by Star did not install new hard drives and Owner’s deliberate decision not to
question the technician. See Guay, 108 F.3d 545 at 551. The markedly low rates and fast service
provided by the technician would provide additional evidence that Owner knew or should have
known that the technician was not installing new hard drives. See Commonwealth v. Franks, 340
A.2d 456, 459–60 (Pa. Super. Ct. 1975) (Price, J., dissenting).
A few states, however, reject this “willful blindness” standard and require actual
knowledge. See Love v. Commonwealth, 55 S.W.3d 816, 825 (Ky. 2001) (actual knowledge of
each element of offense required, but such knowledge may be proved by circumstantial
evidence). In these states, Owner would lack actual knowledge that the technician failed to install
new hard drives and could not be found guilty of false pretenses. See id.
With respect to the second element, a defendant has the intent to defraud required to
establish false pretenses when the defendant intends that the person to whom the false
representation is made will rely upon it. See Commonwealth v. Cheromcka, 850 N.E.2d 1088,
1094 (Mass. App. Ct. 2006). Because Owner made the claim about the new hard drives based
upon the belief that it would help persuade Customer to buy a computer, Owner also had the
requisite intent to defraud.

43
Corporations Analysis

(Corporations V.A.)

Legal Problems: (1) Were the shares owned by Amy on the record date but then sold to
Zach properly voted by Amy or Zach?

(2) Were the shares subject to the proxy given to Dell by Brian
properly voted by Dell, or by Brian, who attended the meeting?

(3) Were the shares held by Carter properly voted by Carter?

(4) Were the shares voted by X Corporation’s president properly


voted?

(5) When the articles of incorporation of a corporation conflict with


the bylaws of a corporation regarding how many shares must vote
in favor of a shareholder proposal, which takes precedence?

DISCUSSION

Summary

Amy’s shares were properly voted by Amy in favor of the proposal even though she no
longer owned the shares when the meeting was held. Zach did not acquire ownership of those
shares until after the record date, and Amy did not grant Zach a proxy to vote those shares at any
time. Brian’s shares were properly voted in favor of the proposal because his earlier proxy to
Dell was revoked when Brian attended the meeting and voted his shares. Carter’s shares were
properly voted against the proposal because he properly attended the meeting and voted them. X
Corporation’s president was not permitted to vote the shares reacquired from Amy.
When the articles of incorporation of a corporation conflict with its bylaws regarding
how many shares must be voted in favor of a shareholder proposal in order for that proposal to
be approved, the articles of incorporation preempt the bylaws. Therefore, the proposal must
receive a two-thirds vote of the outstanding shares entitled to vote on the matter to be approved.
Here, the shareholder proposal received sufficient votes in favor and was approved.

Point One (25%)


Only shareholders of record on the record date are entitled to vote at an annual shareholders’
meeting. Here, Amy was the owner of the 50 shares on the record date, and Zach is not entitled
to vote them. Therefore, Amy’s vote of 50 shares in favor of the shareholder proposal is counted,
and Zach’s vote of 50 shares against the proposal is not.

A record date determines who is entitled to vote at a particular shareholder meeting,


“namely, those persons who were registered as shareholders ‘of record’ on that date.” ROBERT
CHARLES CLARK, CORPORATE LAW § 9.1.2 (1986); Brizzolara v. Sherwood Mem. Park, Inc., 645

44
Corporations Analysis

S.E.2d 508, 518 (Va. 2007). Here, on the record date of December 30th, Amy was the owner of
the 50 shares. Therefore, absent her proxy to another, Amy is entitled to vote the 50 shares. The
fact that Zach bought the shares does not give Zach the right to vote the 50 shares because he did
so after the record date.
[NOTE: If Amy had given Zach a legally binding proxy together with the shares, Zach
might have had the right to vote them even though Zach was not the record owner on the record
date. The facts do not suggest that any proxy was given, however, and therefore Amy’s vote of
50 shares in favor of the proposal should be counted, and Zach’s vote of 50 shares against the
proposal should not be counted.]

Point Two (25%)


Generally, a shareholder proxy is revocable. Any action taken inconsistent with a proxy revokes
that proxy. Here, Brian attended the meeting and voted, which was inconsistent with Brian’s
prior grant of a proxy to Dell. Therefore, Brian effectively revoked the proxy, and his vote of 25
shares in favor of the shareholder proposal should be counted.

Brian gave Dell a proxy to vote Brian’s shares. Shareholder proxies generally are
revocable, and any action inconsistent with the grant of a proxy works as a revocation of that
proxy. See Anish Monga, Using Derivatives to Manipulate the Market for Corporate Control, 12
STAN. J.L. BUS. & FIN. 186, 213 (2006); Richard A. Epstein, Why Restrain Alienation?, 85
COLUM. L. REV. 970, 985 n. 32 (1985) (“Proxies must be revocable unless coupled with an
interest.”); see, e.g., N.Y. BUS. CORP. LAW § 609 (McKinney 1984). A proxy may be made
irrevocable only if the proxy form explicitly so states and the proxy is “coupled with an interest.”
See MODEL BUSINESS CORP. ACT (MBCA) § 7.22(d) (1984 with 2000 amendments) (“An
appointment of a proxy is revocable unless the appointment form or electronic transmission
states that it is irrevocable and the appointment is coupled with an interest.”).
Here, the proxy given by Brian to Dell did not state that it was irrevocable, nor was it
coupled with an interest, and therefore it was revocable. Brian’s attendance at the shareholders’
meeting and his vote of the 25 shares were actions inconsistent with the grant of a proxy
covering those shares to Dell. Therefore, Brian effectively revoked the proxy, and Brian’s vote
of 25 shares in favor of the proposal should be counted while Dell’s vote of 25 shares against the
proposal should not be counted.

Point Three (10%)


A shareholder of record may attend and vote at an annual shareholders’ meeting. Thus Carter’s
vote of 25 shares will be counted against the proposal.

A shareholder of record is entitled to attend and vote at an annual shareholders’ meeting


unless he or she has executed a valid, irrevocable proxy covering his shares. See MBCA
§ 7.22(a). Here, Carter attended the meeting and properly voted his 25 shares against the
proposal.

Point Four (20%)


Shares that are repurchased by a corporation are not outstanding shares and therefore may not be
voted. Therefore, the vote of 50 shares by X Corporation’s president against the proposal cannot
be counted.

45

Corporations Analysis

Shares that are reacquired by a corporation are considered authorized but not outstanding.
MBCA § 6.31(a). Some jurisdictions refer to these as “treasury shares.”
In counting shareholder votes, “each outstanding share . . . is entitled to one vote on each
matter voted on at a shareholders’ meeting.” Id. § 7.21(a). The facts indicate that X
Corporation’s president voted 50 shares that the corporation reacquired from Amy. Because
reacquired shares are not outstanding, they cannot be voted. Therefore the 50 shares voted by the
president do not count.

Point Five (20%)


When the articles of incorporation conflict with a corporation’s bylaws regarding how many
shares must vote in favor of a shareholder proposal in order for that proposal to be approved, the
articles of incorporation prevail over the bylaws. Here, the shareholder proposal received
sufficient votes in favor and was approved.

When a corporation’s articles of incorporation conflict with its bylaws regarding how
many shares must vote in favor of a shareholder proposal in order for that proposal to be
approved, the articles of incorporation preempt the bylaws. See Boatmen’s First Nat’l Bank of
West Plains v. Southern Mo. Dist. Council of the Assemblies of God, 806 S.W.2d 706, 713 (Mo.
Ct. App. 1991) (“[A]ny bylaw provision that conflicts with the articles of incorporation is
void.”). Here, X Corporation’s Articles of Incorporation requiring approval of two-thirds of the
shares outstanding and entitled to vote on the matter to approve any shareholder action will
preempt the bylaws’ requirement of unanimity.
Here, Amy’s 50 votes and Brian’s 25 votes in favor of the proposal are counted, while
only Carter’s 25 votes against the proposal are counted. Therefore, the proposal received the
necessary approval by two-thirds of the shares outstanding and entitled to vote on the matter and
should pass.

46

Das könnte Ihnen auch gefallen